Answer: x = 15
Step-by-step explanation:
Given:
27 + x = 42
Subtract 27 from both sides of the equation:
x = 15
27 + x = 42
Subtract 27 from both sides:
X = 15
What is the surface area of a sphere with a radius of 15 cm?
Answer:
2827.43 cm^2
Step-by-step explanation:
Hope this helped! <3 Have a great day :) Brainliest please?
57771298
How would the mode change if the number 5 were added to the set?
Answer:
The mode of this list of numbers would still be [tex]7[/tex] (the mode did not change) after "[tex]5[/tex]" was added to the list.
Step-by-step explanation:
The mode of a list of elements is the member that appears the most frequently.
The original list of numbers in this question [tex][5,\, 7,\, 7,\, 7,\, 1,\, 2,\, 9,\, 8][/tex] included the following members:
[tex]\begin{array}{c|c} \text{Member} & \text{Number of appearances} \\ \hline\hline 1 & 1 \\\hline 2 & 1 \\\hline 5 & 1\\\hline 7 & 3 \\\hline 8 & 1 \\\hline 9 & 1 \\\end{array}[/tex]
[tex]\begin{array}{c|c} \text{Member} & \text{Number of appearances} \\ \cline{1-2}\\[-1em]\cline{1-2} 1 & 1 \\\cline{1-2} 2 & 1 \\\cline{1-2} 5 & 1\\\cline{1-2} 7 & 3 \\\cline{1-2} 8 & 1 \\\cline{1-2} 9 & 1 \\\end{array}[/tex].
The member "[tex]7[/tex]" appeared more often than any other member of this list. Thus, the mode of the original list was "[tex]7\![/tex]".
Adding "[tex]5[/tex]" to the original list would give the list [tex][5,\, 7,\, 7,\, 7,\, 1,\, 2,\, 9,\, 8,\, 5][/tex]. Members of this list are:
[tex]\begin{array}{c|c} \text{Member} & \text{Number of appearances} \\ \hline\hline 1 & 1 \\\hline 2 & 1 \\\hline 5 & 2\\\hline 7 & 3 \\\hline 8 & 1 \\\hline 9 & 1 \\\end{array}[/tex]
[tex]\begin{array}{c|c} \text{Member} & \text{Number of appearances} \\ \cline{1-2}\\[-1em]\cline{1-2} 1 & 1 \\\cline{1-2} 2 & 1 \\\cline{1-2} 5 & 2\\\cline{1-2} 7 & 3 \\\cline{1-2} 8 & 1 \\\cline{1-2} 9 & 1 \\\end{array}[/tex].
The member "[tex]7[/tex]" still appeared more often than any other member of this list. Thus, the mode of the new list would also be "[tex]7\![/tex]".
A cylinder has a radius 3 inches and height 5 inches. A cone has the same radius and height. What is the volume of the cylinder
✰ Given Information :-
⠀
A cylinder with following dimensions :
Radius = 3 inchesHeight = 5 inches⠀
✰ To Find :-
⠀
The volume of the cylinder⠀
✰ Formula Used :-
⠀
[tex] \qquad \star \: \underline{ \boxed{ \purple{\sf Volume_{Cylinder} = \pi {r}^{2} h}}} \: \star[/tex]
⠀
Where,
r = radius h = height⠀
✰ Solution :-
⠀
Putting the values in the formula, we get,
⠀
[tex] \sf \longrightarrow Volume=3.14 \times {(3)}^{2} \times 5 \\ \\ \\ \sf \longrightarrow Volume=3.14 \times 45 \: cm \: \: \: \: \: \\ \\ \\ \sf \longrightarrow \underline{ \boxed{ \green{ \frak{Volume= {141 .3 \: cm}^{3} }}}} \: \star \: \: \: \: \: \\ [/tex]
⠀
Thus, the volume of the cylinder is 141.3 cm³.
⠀
[tex] \underline{ \rule{230pt}{2pt}} \\ \\ [/tex]
Step-by-step explanation:
As it is given that, a cylinder has radius 3 inches and height 5 inches and we are to find the volume of the cylinder, also a extra information is given that is a cone has the same radius and height.
[tex] \: [/tex]
We know,
[tex]{ \longrightarrow\qquad{\frak {\pmb{Volume_{(cylinder) } = \pi {r}^{2}h }}}} \\ \\[/tex]
Where,
r is the radius of the cylinder.h is the height of the cylinder.Here, we will take the value of π as 3.14 approximately .[tex] \: [/tex]
Now, we will substitute the given values in the formula :
[tex] \: [/tex]
[tex]{ \longrightarrow\qquad{\sf {\pmb{Volume_{(cylinder) } = 3.14 \times ( {3})^{2} \times 5 }}}} \\ \\[/tex]
[tex]{ \longrightarrow\qquad{\sf {\pmb{Volume_{(cylinder) } = 3.14 \times 9 \times 5 }}}} \\ \\[/tex]
[tex]{ \longrightarrow\qquad{\sf {\pmb{Volume_{(cylinder) } = 3.14 \times 45 }}}} \\ \\[/tex]
[tex]{ \longrightarrow\qquad{\frak {\pmb{Volume_{(cylinder) } = 141.3 }}}} \\ \\[/tex]
Note :
Answer might be different if we take the value of π as 22/7 .In a writing competition, the first place winner receives 1/2 of the prize money. The second runner up receives 1/4 of what the winner won. What was the total amount of prize money distributed if the winner receives $8000
Answer:
10,000 is the total amount of.money that was distributed
[tex]\begin{equation}\text { Question: If } \ln (x+y)=4 \times y \text {. Find } \frac{d^{2} y}{d x^{2}} \text { at } x=0 \text {. }\end{equation}[/tex]
I think you meant to say
[tex]\ln(x+y) = 4xy[/tex]
and not "4 times y" on the right side (which would lead to a complex value for y when x = 0). Note that when x = 0, the equation reduces to ln(y) = 0, so that y = 1.
Implicitly differentiating both sides with respect to x, taking y = y(x), and solving for dy/dx gives
[tex]\dfrac{1+\frac{dy}{dx}}{x+y} = 4y + 4x\dfrac{dy}{dx}[/tex]
[tex]\implies \dfrac{dy}{dx} = \dfrac{4xy+4y^2-1}{1-4x^2-4xy}[/tex]
Note that when x = 0 and y = 1, we have dy/dx = 3.
Differentiate both sides again with respect to x :
[tex]\dfrac{d^2y}{dx^2} = \dfrac{(1-4x^2-4xy)\left(4y+4x\frac{dy}{dx}+8y\frac{dy}{dx}\right)-(4xy+4y^2-1)\left(-8x-4y-4x\frac{dy}{dx}\right)}{(1-4x^2-4xy)^2}[/tex]
No need to simplify; just plug in x = 0, y = 1, and dy/dx = 3 to get
[tex]\dfrac{d^2y}{dx^2} \bigg|_{x=0} = \boxed{40}[/tex]
An open box is to be made from a square piece of cardboard, 18 inches by 18 inches, by removing a small square from each corner and folding up the flaps to form the sides. What are the dimensions of the box of greatest volume that can be constructed in this way
The dimension of the box of the greatest volume that can be constructed in this way is 12x12x3 and the volume is 432.
How to solve the dimension?
Let x be the side of the square to remove. Then the volume of the box is:
V(x) = (18 - 2x)² * x = 324x - 72x² + 4x³
To find the maximum volume, differentiate and set it to 0:
V'(x) = 324 - 144x + 12x²
0 = x² - 12x + 27
0 = (x - 9)(x - 3)
x = 3 or 9
When x = 3,
V"(x) =-144+24x
V"(3) =-144+72=-72<0
so volume is maximum at x=3
Therefore the box is 12x12x3 and the volume is 432.
Learn more about dimension on:
https://brainly.com/question/26740257
The circumference of a circle is 17π in. What is the area, in square inches? Express your answer in terms of π?
Answer:
22.97
Step-by-step explanation:
17 divided by pi is 5.41
5.41 divided by 2 is 2.075
r^2(pi)
is 2.075 x 2.075 x 3.41 is 22.97
Brennan has been playing a game where he can create towns and help his empire expand. Each town he has allows him to create 1.15 times as many villagers as he had in the one before. The game gave Brennan 5 villagers to start with. Explain to Brennan how to create an equation to predict the number of villagers in any specific town. Then show how to use your equation to solve for the number of villagers he can create to live in the 15th town.
Answer:
Use the geometric sequence formula since you are multiplying each town's villagers by the amount your teacher gave you.
Step-by-step explanation:
Plug in the parts of the formula from the numbers given in the question, then you can calculate the number of villagers needed for the 15th town.
Answer:
35.375 Villagers Live in the 15th village
Step-by-step explanation:
Brennan can use the Geometric Sequence formula to predict the number of villagers in the 15th town.
(work attached)
solve for x ~
[tex]4x - 16 = 64[/tex]
ty ^^
[tex]\qquad\qquad\huge\underline{{\sf Answer}}♨[/tex]
Let's solve for x ~
[tex]\qquad \sf \dashrightarrow \:4x - 16 = 64[/tex]
[tex]\qquad \sf \dashrightarrow \:4(x - 4) = 64[/tex]
[tex]\qquad \sf \dashrightarrow \:(x - 4) = 64 \div 4[/tex]
[tex]\qquad \sf \dashrightarrow \:x - 4 = 16[/tex]
[tex]\qquad \sf \dashrightarrow \:x = 16 + 4[/tex]
[tex]\qquad \sf \dashrightarrow \:x =20[/tex]
[tex] \boxed{ \bf \huge \: Answer \: }[/tex]
[tex] \: \: [/tex]
[tex] \sf \large \: 4x - 16 = 64[/tex]
[tex] \: \: [/tex]
[tex] \sf \large \: 4x = 64 + 16[/tex]
[tex] \: \: [/tex]
[tex] \sf \large \: 4x = 40[/tex]
[tex] \: [/tex]
[tex] \sf \large \: x = \cancel \frac{80}{4} [/tex]
[tex] \: \: [/tex]
[tex] \sf \large x = 20[/tex]
[tex] \: \: [/tex]
Hope It's Helps uh Miss<3"
a 5 1/2 quart pot is filled 2/3 of the way with water. How many quarts of water can the pit hold?
Answer:
4 5/6 quarts or 4.83 quarts
Step-by-step explanation:
5 1/2 quart pot has an equal value to 11/2 quart pot
it is filled with 2/3 quarts of water
Therefore, the quarts of water the pot can hold is;
11/2 - 2/3
LCM = 6
11/2 - 2/3 ÷ 6
33 - 4 ÷ 6
29/6
:. 29/6 quarts(4 5/6 quarts or 4.83 quarts) of water is the amount needed to fill the 5 1/2 quart pot
Question 3
Compute the probability of Susan turning 44 years old (event A), given that Carl has reached the age of 44 (event B).
Answer:
Events A and B are independent of each other, so P(B∣A)=P(B)=0.95726.
Explanation -
I got this answer from Plato.
What will the balance be after 10 years if $1500 is invested at 1.5% interest compounded continuously?
Answer:
A = $1,742.75
A = P + I
Where
P is the principal (1,500.00)
I is the interest (242.75)
Steps:
First, convert R as a percent to r as a decimal
r = R/100
r = 1.5/100
r = 0.015 rate per year,
Then solve the equation for A
[tex]A = Pe^{rt}\\A = 1,500.00(2.71828)^{(0.015)(10)}\\A = $1,742.75[/tex]
Summary:
The total amount with compound interest on a principal of $1,500.00 at a rate of 1.5% per year compounded continuously over 10 years is $1,742.75.
pls help me uwuwuwuuwuwuww
Answer:
128
6
12
48
Step-by-step explanation:
solve and ( SHOW YOUR WORK !! )
Answer:
yea
Step-by-step explanation:
The length of a rectangle multilayered by 3 is equal to 4 times its width. The perimeter is 8 2/5 feet. Find the length and the width.
The required length of the rectangle is 12/5 feet and the width is 9/5 feet.
What is the equation model?The equation model is defined as the model of the given situation in the form of an equation using variables and constants.
Here,
Let's denote the length of the rectangle as L and the width as W.
From the problem statement, we know that:
3L = 4W (equation 1)
We also know that the perimeter P is given by:
P = 2L + 2W = 8 2/5 feet
Multiplying both sides by 5 to get rid of the fraction, we get:
10L + 10W = 42
Dividing by 2, we get:
5L + 5W = 21 (equation 2)
Now we can use equations 1 and 2 to solve for L and W.
First, we can substitute equation 1 into equation 2 to get an equation in terms of W:
5L + 15L/4 = 21
Multiplying both sides by 4, we get:
20L + 15L = 84
35L = 84
L = 84/35 = 12/5 feet
Now we can substitute the value of L into equation 1 to get the value of W:
3L = 4W
3(12/5) = 4W
W = 9/5 feet
Therefore, the length of the rectangle is 12/5 feet and the width is 9/5 feet.
Learn more about models here:
https://brainly.com/question/22591166
#SPJ7
It is estimated that 60% of U.S. households subscribe to cable TV. You would like to verify this statement for your class in mass communications. If you want your estimate to be within 5 percentage points, with a 95% level of confidence, how many households should you sample
Using the z-distribution, as we are working with a proportion, it is found that you should sample 577 households.
What is a confidence interval of proportions?A confidence interval of proportions is given by:
[tex]\pi \pm z\sqrt{\frac{\pi(1-\pi)}{n}}[/tex]
In which:
[tex]\pi[/tex] is the sample proportion.z is the critical value.n is the sample size.The margin of error is given by:
[tex]M = z\sqrt{\frac{\pi(1-\pi)}{n}}[/tex]
In this problem, we have a 95% confidence level, hence[tex]\alpha = 0.95[/tex], z is the value of Z that has a p-value of [tex]\frac{1+0.95}{2} = 0.975[/tex], so the critical value is z = 1.96.
In this problem, the estimate is of [tex]\pi = 0.6[/tex], and we want a margin of error of M = 0.05, hence we solve for n to find the sample size.
[tex]M = z\sqrt{\frac{\pi(1-\pi)}{n}}[/tex]
[tex]0.05 = 1.96\sqrt{\frac{0.6(0.4)}{n}}[/tex]
[tex]0.05\sqrt{n} = 1.96\sqrt{0.6(0.4)}[/tex]
[tex]\sqrt{n} = \frac{1.96\sqrt{0.6(0.4)}}{0.05}[/tex]
[tex](\sqrt{n})^2 = \left(\frac{1.96\sqrt{0.6(0.4)}}{0.05}\right)^2[/tex]
[tex]n = 576.24[/tex]
Rounding up, you should sample 577 households.
More can be learned about the z-distribution at https://brainly.com/question/25890103
Using the three Indicated points on the given line and the concept of similar triangles, show and explain why the slope of the line is the same between any two of these three points. calculate the slope of the line
Answer:
it is the same because slope is a measure of steepness.
Step-by-step explanation:
(0,1) (2,4)
4-1/2-0=3/2
slope=3/2
3/4 of tom's favorite number is 36. what number is one half of tom's favorite number
Answer: 3/4 is 75%
Step-by-step explanation: Since his fav number is 36 the half of 36 is 18 bc 18 + 18 is 36
If f is a second-degree polynomial function such that f(3)=5, f'(3)=6, and f''(3)=4, what is the value of f(2) ?
so let's say the equation is y = ax² + bx + c, where a,b,c are constants.
we know that f(3) = 5, or namely when x = 3 , y = 5.
we also know that f'(3) = 6, or namely the slope at the point (3,5) is 6.
we also know that at (3,5) the 2nd derivative is 4, so a positive number simply tell us its concavity, is up, so is a parabola opening upwards.
[tex]y=ax^2+bx+c\implies \left. \cfrac{dy}{dx}=2ax+b \right|_{x=3}~~ = ~~ 6\implies 6=2ax+b \\\\\\ 6=2a(\stackrel{x}{3})+b\implies 6=6a+b\implies 6-6a=b\implies \boxed{6(1-a)=b} \\\\[-0.35em] ~\dotfill\\\\ \left. \cfrac{d^2y}{dx^2}=2a \right|_{x=3}~~ = ~~4\implies 2a=4\implies a=\cfrac{4}{2}\implies \boxed{a=2}~\hfill \boxed{-6=b} \\\\[-0.35em] ~\dotfill[/tex]
[tex]y=2x^2-6x+c\qquad \begin{cases} x=3\\ y = 5 \end{cases}\implies 5=2(3)^2-6(3)+c\implies \boxed{5=c} \\\\\\ y=2x^2-6x+5~\hfill y(2)=2(2)^2-6(2)+5\implies \blacktriangleright y(2)=1 \blacktriangleleft[/tex]
5 chocolate bars cost $5.75
2 chocolate bars and 3 packets of sweets cost $7.85 work out the cost of one packet of sweets
Answer:
The answer is $1.85
Step-by-step explanation:
Given;The cost of 5 chocolate bars = $5.75The cost of 2 chocolate bars and 3 packets of sweets = $7.85To Find;The cost of one packet of sweets.Now, we know that the cost of 5 chocolate bars are $5.75.
For 1 chocolate bar
5.75 ÷ 5 = $1.15
Here, we get the price of 1 chocolate bar
So, the cost of 2 chocolate bars and 3 packets of sweets are $7.85.
2 × chocolate bar = 2 × (1.15) = $2.3
Now, we want to find only the cost of one packet of sweets.
So,
7.85 – 2.3 = $5.55
3 packets of sweets cost $5.55
For 1 packets of sweets
5.55 ÷ 3 = $1.85
Thus, The cost of one packet of sweets = $1.85
I need extra help. Someone
Answer:
C. tanS = 5/12, tanT = 12/5
Step-by-step explanation:
Tangent has NOTHING to do with hypotenuse, do we can easily eliminate answer choices involving 13, the hypotenuse. For angle S, the opposite side measures 5 and the adjacent side measures 12. For angle T, the opposite side measures 12 and the adjacent side measures 5. See this below algebraically
tanθ = opp/adj
tanS = 5/12
tanT = 12/5
I hope this helps!
17,18,22,31,47, ... what's next
Answer:
Step-by-step explanation:
First establish the pattern. Notice the sequence of numbers below. These are the ones making up the pattern. You can go as far as 47 - 31 which gives 16 which is 4^2.
18 -17 = 1 = 1^2
22 - 18 = 4 = 2^2
31 - 22 = 9 = 3^2
47 - 31 = 16 = 4^2
=================
Call the number comming x. 47 taken from x must give 5^2 which is 25
x - 47 = 5^2
x - 47 = 25 Add 47 to both sides
x -47 + 47 = 25 + 47
x = 72
=================
72 -47 = 25 which is 5^2.
Answer: The next number is 72
Please help!!! I'm struggling
Step-by-step explanation:
please mark me as brainlest
Answer:
I hope it helped you
Step-by-step explanation:
Pls refer the given attachment
[tex]1. ) \: \frac{4}{5} + \frac{2}{3} = [/tex]
[tex]2. )4 \: \frac{8}{12} + 3 \frac{1}{6} =[/tex]
[tex]3. \frac{13}{17} + \frac{3}{4} = [/tex]
[tex]4. 23 \frac{1}{5} - 7 \frac{1}{4} =[/tex]
[tex] \\ [/tex]
With Solution!!
Answer:
1. 4/5+2/3=?
The fractions have unlike denominators. First, find the Least Common Denominator and rewrite the fractions with the common denominator.
LCD(4/5, 2/3) = 15
Multiply both the numerator and denominator of each fraction by the number that makes its denominator equal the LCD. This is basically multiplying each fraction by 1.
(45×33)+(23×55)=?
Complete the multiplication and the equation becomes
12/15+10/15=?
The two fractions now have like denominators so you can add the numerators.
Then: 12+10/15=22/15
This fraction cannot be reduced. The fraction
22/15 is the same as 22÷15
Convert to a mixed number using long division for 22 ÷ 15 = 1R7, so
22/15=17/15
Therefore: 4/5+2/3=17/15
Solution by Formulas Apply the fractions formula for addition, to 4/5+2/3 and solve
(4×3)+(2×5)/5×3
=12+10/15
=22/15 Convert to a mixed number using long division for 22 ÷ 15 = 1R7, so 22/15=17/15 Therefore:4/5+2/3=1 7/15
Step-by-step explanation:
Answer:
See below.
Step-by-step explanation:
1.
4/5 + 2/312/15 + 10/1522/15 or 1 7/152.
4 8/12 + 3 1/64 2/3 + 3 1/614/3 + 19/628/6 + 19/647/6 or 7 5/63.
13/17 + 3/452/68 + 51/68103/68 or 1 35/684.
23 1/5 - 7 1/4116/5 - 29/423.2 - 7.2515.95319/20 or 15 19/20dude why is no one answering. Michelle walks 1/4
hour every day of the week (Monday through Sunday). Sandra walks 2/4
hour per day, but only on weekdays (Monday through Friday). Both Michelle and Sandra continue their routines for 9 weeks.
Calculate who walked the most over this 9 week period, and calculate how much more time this person walked than the other person. Be sure to show how you calculated each person's total walking time as a mixed fraction, and show how you calculated how much more time was walked by either Michelle or Sandra as a mixed fraction.
Answer:
Michelle walks 1 4 hour every day of the week (Monday through Sunday). Sandra walks 2 4 hour per day, but only on weekdays (Monday through Friday). Both Michelle and Sandra continue their routines for 9 weeks. Calculate who walked the most over this 9 week period, and calculate how much more time this person walked than the other person. Be sure to show how you calculated each person's total walking time as a mixed fraction, and show how you calculated how much more time was walked by either Michelle or Sandra as a mixed fraction. this is a word problom
Answer:
people are not doing it it's because that it's multiplying mixed numbers by whole numbers and we don't really need to do that
Step-by-step explanation:
The mean of the ages of 5 brothers is 13 years.
12, 16, O, 14, 8
Answer:
O = 15
Step-by-step explanation:
.mean ' is the average
(12 + 16 + O + 14 + 8) / 5 = 13
50 + O = 65
O = 15
a] The area of a square is 200 cm² Find the length of its diagonal.
[tex]"20cm"[/tex]
Step-by-step explanation:[tex]Area\ of\ rq=a^2[/tex]
[tex]a=\sqrt{200}=10\bullet \sqrt{2}[/tex]
[tex]d\ iargul=\sqrt{2}9[/tex]
[tex]=10\times \sqrt{2}\times \sqrt{2}[/tex]
[tex]=20cm[/tex]
I hope this helps you
:)
Help asap for brainliest
Answer:
282 ft
Step-by-step explanation:
Area of 2 triangles = 6+6
= 12 ft
Area of 3 rectangles = 90+90+90
= 270 ft
therefore
the surface area = 12 + 270
= 282 ft
I think that I am right
What is the value of the expression below when x=4 and y=3?
The question is 5x to the 3rd power plus 4y to the 3rd power
[tex]x = 4\\y = 3[/tex]
Question is [tex]5x^3 + 4y^3[/tex]
We know what [tex]x[/tex] and [tex]y[/tex] are.
[tex]5x^3 + 4y^3 = 5 \times 4^3 + 4 \times 3^3\\= 5 \times 64 + 4 \times 27\\= 320 + 108\\= 428[/tex]
solve .-. ~
[tex] \\ \\x - \frac{1}{2} \geqslant 43 \\ [/tex]
thankyou .-.
[tex]\qquad\qquad\huge\underline{{\sf Answer}}♨[/tex]
Let's solve ~
[tex]\qquad \tt \dashrightarrow \:x - \dfrac{1}{2} \geqslant 43 [/tex]
[tex]\qquad \tt \dashrightarrow \:x \geqslant43 + \dfrac{1}{2} [/tex]
[tex]\qquad \tt \dashrightarrow \:x \geqslant \dfrac{86 + 1}{2} [/tex]
[tex]\qquad \tt \dashrightarrow \:x \geqslant \dfrac{87}{2} [/tex]
hope this helps Belinda aunty ~